Difference between revisions of "1999 AIME Problems/Problem 9"
(solution) |
m (→Solution: typos) |
||
Line 3: | Line 3: | ||
== Solution == | == Solution == | ||
− | Suppose we pick an arbitrary point on the line <math>x = y</math> on the [[complex plane]], say <math>(1,1)</math>. According to the definition of <math>f(1+i) = (a+bi)(1+i) = (a-b) + (a+b)i</math>, the image must be equidistant to <math>(1,1)</math> and <math>(0,0)</math>. These points lie on the line with slope <math>-1</math> and which passes through <math>\left\frac 12, \frac12\right)</math>, so its graph is <math>x + y = 1</math>. Substituting <math>x = (a-b)</math> and <math>y = (a+b)</math>, we get <math>2a = 1 \Rightarrow a = \frac 12</math>. | + | Suppose we pick an arbitrary point on the line <math>x = y</math> on the [[complex plane]], say <math>(1,1)</math>. According to the definition of <math>f(z) = f(1+i) = (a+bi)(1+i) = (a-b) + (a+b)i</math>, the image must be equidistant to <math>(1,1)</math> and <math>(0,0)</math>. These points lie on the line with slope <math>-1</math> and which passes through <math>\left(\frac 12, \frac12\right)</math>, so its graph is <math>x + y = 1</math>. Substituting <math>x = (a-b)</math> and <math>y = (a+b)</math>, we get <math>2a = 1 \Rightarrow a = \frac 12</math>. |
By the [[Pythagorean Theorem]], we have <math>\left(\frac{1}{2}\right_^2 + b^2 = 8^2 \Longrightarrow b^2 = \frac{255}{4}</math>, and the answer is <math>\boxed{259}</math>. | By the [[Pythagorean Theorem]], we have <math>\left(\frac{1}{2}\right_^2 + b^2 = 8^2 \Longrightarrow b^2 = \frac{255}{4}</math>, and the answer is <math>\boxed{259}</math>. |
Revision as of 15:43, 18 October 2007
Problem
A function is defined on the complex numbers by where and are positive numbers. This function has the property that the image of each point in the complex plane is equidistant from that point and the origin. Given that and that where and are relatively prime positive integers. Find
Solution
Suppose we pick an arbitrary point on the line on the complex plane, say . According to the definition of , the image must be equidistant to and . These points lie on the line with slope and which passes through , so its graph is . Substituting and , we get .
By the Pythagorean Theorem, we have $\left(\frac{1}{2}\right_^2 + b^2 = 8^2 \Longrightarrow b^2 = \frac{255}{4}$ (Error compiling LaTeX. Unknown error_msg), and the answer is .
See also
1999 AIME (Problems • Answer Key • Resources) | ||
Preceded by Problem 8 |
Followed by Problem 10 | |
1 • 2 • 3 • 4 • 5 • 6 • 7 • 8 • 9 • 10 • 11 • 12 • 13 • 14 • 15 | ||
All AIME Problems and Solutions |